Piecewise initial condition heat equation

Click For Summary
The discussion focuses on solving the heat equation with a piecewise initial condition defined by a function that is zero in two intervals and 100 in between. The coefficients for the Fourier series solution are derived using the formula, revealing that all even indices result in zero. Concerns are raised about the accuracy of the graph at t=0, suggesting that insufficient terms may be used to approximate the piecewise function effectively. Additionally, it is noted that achieving a local maximum temperature at x=π/2 for t>0 requires careful consideration of the number of terms in the series. The analysis concludes that abrupt changes in the initial condition necessitate more terms for a smoother approximation.
raditzan
Messages
1
Reaction score
0

Homework Statement


I have the solution to the heat equation, with the BC's and everything but the IC applied. So I am just trying to solve for the coefficients, the solution without the coefficients is
$$u(x,t) = \sum_{n=1}^{\infty} A_n\sin(nx)e^{-n^2t}$$
If the initial condition is ##u(x,0) = f(x)## such that $$f(x) = \begin{cases}
0 & 0 < x < \frac{\pi}{3} \\
100 & \frac{\pi}{3} < x < \frac{2\pi}{3} \\
0 & \frac{2\pi}{3} < x < \pi
\end{cases}
$$
I used the formula $$A_m = \frac{2}{\pi}\int_0^\pi f(x)\sin(mx)dx=\frac{200}{m\pi}\bigg[\cos(\frac{\pi}{3}m) - cos(\frac{2\pi}{3}m)\bigg]$$

I couldn't find a pattern in the coefficients other than all the even indices go to ##0##. Also is this even correct? When I try to graph this at ##t=0## it isn't giving me the piecewise function ##f(x)##. Is it just that I didn't use enough terms to make it noticeable? Also how would I show that at any time ##t>0## the temperature distribution in the rod achieves a local maximum at ##x=\pi/2##?
 
Last edited:
Physics news on Phys.org
That's right. Use more terms. I used 100, and it was still pretty rough. Fourier sums like your solution work well for smooth functions, abrupt changes like that require a lot of terms to reproduce.
 
raditzan said:
I used the formula $$A_m = \frac{2}{\pi}\int_0^\pi f(x)\sin(mx)dx=\frac{200}{m\pi}\bigg[\cos(\frac{\pi}{3}m) - cos(\frac{2\pi}{3}m)\bigg]$$

I couldn't find a pattern in the coefficients other than all the even indices go to ##0##. Also is this even correct? When I try to graph this at ##t=0## it isn't giving me the piecewise function ##f(x)##. Is it just that I didn't use enough terms to make it noticeable? Also how would I show that at any time ##t>0## the temperature distribution in the rod achieves a local maximum at ##x=\pi/2##?

$$\cos(\frac{\pi}{3}m) - cos(\frac{2\pi}{3}m)=\cos\left(\frac{\pi}{3}m\right)-2\cos^2\left(\frac{\pi}{3}m\right)+1$$
m=1: ##\frac{1}{2}-\frac{1}{2}+1=1##
m = 2: ##-\frac{1}{2}-\frac{1}{2}+1=0##
m=3: ##-1-2+1=-2##
m=4: ##-\frac{1}{2}-\frac{1}{2}+1=0##
m=5: ##1##
m=6: ##0##
m=7: ##1##
m=8: ##0##
m=9: ##-2##

There are 3 separate summations.

For m=6k+1 (k=0,1,2,...), the coeff is 1
For m = 6k+3 (k=0,1,2,..), the coeff is -2
For m = 6k+5 (k=0,1,2,...), the coeff is 1
 
Question: A clock's minute hand has length 4 and its hour hand has length 3. What is the distance between the tips at the moment when it is increasing most rapidly?(Putnam Exam Question) Answer: Making assumption that both the hands moves at constant angular velocities, the answer is ## \sqrt{7} .## But don't you think this assumption is somewhat doubtful and wrong?

Similar threads

  • · Replies 11 ·
Replies
11
Views
3K
  • · Replies 14 ·
Replies
14
Views
2K
  • · Replies 5 ·
Replies
5
Views
2K
  • · Replies 2 ·
Replies
2
Views
2K
Replies
3
Views
2K
  • · Replies 6 ·
Replies
6
Views
2K
  • · Replies 5 ·
Replies
5
Views
1K
Replies
9
Views
2K
  • · Replies 1 ·
Replies
1
Views
1K
  • · Replies 7 ·
Replies
7
Views
2K